LSAT and Law School Admissions Forum

Get expert LSAT preparation and law school admissions advice from PowerScore Test Preparation.

User avatar
 Dave Killoran
PowerScore Staff
  • PowerScore Staff
  • Posts: 5853
  • Joined: Mar 25, 2011
|
#27055
Complete Question Explanation
(The complete setup for this game can be found here: lsat/viewtopic.php?t=3418)

The correct answer choice is (B)

If M is not selected to attend the retirement dinner, from our analysis of the groups we can infer that L and Q must both be selected. Of course, when Q is selected then K must be selected, leading to the following setup:
Oct 03_M12_game#2_L5_explanations_game#3_#11_diagram_1.png
Accordingly, answer choice (B) is correct.
You do not have the required permissions to view the files attached to this post.
 lorein21
  • Posts: 17
  • Joined: Sep 30, 2011
|
#2148
Hi I was hoping someone could help me understand the question below.


for 11 - I see why K and L have to exist (answer choice b) but when I remove L and P (answer choice D) and N and Q (answer choice E) I still cannot create a set of 4 that could work. Am I misunderstanding the question? I just don't see why the answer has to be B.
User avatar
 Dave Killoran
PowerScore Staff
  • PowerScore Staff
  • Posts: 5853
  • Joined: Mar 25, 2011
|
#2149
Hi Lorein,

Let's take a look at your question and see if we can figure this out.

Question #11: To answer this question, let's go back into the setup of the game and examine one of the inferences. Because J, K, N, and P will collectively occupy exactly two of the spaces (this is known from the first two rules), the remaining two spaces are occupied by the group of L, M, and Q. Thus, we can infer that two people from the group of L, M, and Q must always be selected. Note that any time one of the members of the group of L, M, and Q is not selected, the other two must be selected.

Now, keep in mind that the question stem removes M from further consideration. If M is not selected to attend the retirement dinner, from our analysis above we can infer that L and Q must both be selected. Of course, from the fourth rule, when Q is selected then K must be selected, meaning that L, Q, and K must always be selected in this question. The only other choice is for the fourth person in the group, and that choice is either N or P. Thus, the following group is available in this question:

L, Q, K, N/P

Thus, only two possible solutions exist in this question:

1. L, Q, K, N
or
2. L, Q, K, P

So, let's look at the two answer choices you ask about, and see if they contain a pair of people both of whom must be selected.

Answer choice (D): Do both L and P have to be selected? No, they can be, but this is a Must question and one of our scenarios does not include both L and P.

Answer choice (E): Do both N and Q have to be selected? No, they can be, but this is a Must question and one of our scenarios does not include both N and Q.


Please let me know if the above helps explain this question. Thanks!
 lorein21
  • Posts: 17
  • Joined: Sep 30, 2011
|
#2188
Thanks for the help!
 lorettan102
  • Posts: 11
  • Joined: Nov 02, 2016
|
#30260
I realize now in doing this diagram set-up incorrectly that the correct answer choice is easy to find when the diagram is correct! I could kick myself for that!
 beeke
  • Posts: 4
  • Joined: Jan 15, 2018
|
#42875
I cannot figure out why answer choice A is wrong. I see why B can be the correct answer, but I also see how A can be the correct answer.

Without M, then both L and Q must be selected because the diagram is: 1. (J/K) 2. (N/P) 3 - 4. (LMQ). Without M, then Q and L must be selected (answer choice A). It would then be K (N/P) L Q. L and Q and K are all in the answer choices. What makes B right and not A?
 Jon Denning
PowerScore Staff
  • PowerScore Staff
  • Posts: 904
  • Joined: Apr 11, 2011
|
#42989
Hi beeke - thanks for the question, and welcome to the Forum! I think this is a simple one to resolve, as it appears you've just misread answer choice A :)

Answer choice A contains J and Q both in, NOT L and Q. But J and Q can never be selected together, since Q brings in K, and K knocks out J. So I suspect this one was down to a small oversight on your end!

For anyone else wondering more about this question, however, let me take a moment to explain it and make sure all is clear!

First, we know that if M is out we have this situation to start us off:

..... In: J/K , N/P , Q , L

..... Out: K/J , P/N , M

And from the final rule we know that Q :arrow: K, meaning that J/K split is actually resolved here, with K in (because Q is in) and J out. So we can adjust the info above accordingly:

..... In: N/P , K , Q , L

..... Out: P/N , J , M

And now hopefully you can see the problem with the four wrong answers: they either present a variable that cannot be in (like J in answer choice A), or contain a variable that does not have to be in (either N or P, like answers C, D, and E).
 carolmorales
  • Posts: 3
  • Joined: Oct 30, 2020
|
#81531
Hello everyone! I selected B for this one and understand why it's the correct choice. However, answer E confused me. If M is out, L and Q must stay. The third rule tells me that N cannot be selected unless L is selected, and since we have to keep L, I kicked P out. And came up with:

K N L Q

Can someone explain why both N and P can stay? Thanks!
 Robert Carroll
PowerScore Staff
  • PowerScore Staff
  • Posts: 1787
  • Joined: Dec 06, 2013
|
#81576
Carol,

Be careful! You made a Mistaken Reversal. "N cannot be selected unless L is selected" is a conditional. Use the Unless Equation! Whatever is modified by "unless" is necessary - so L is necessary. The rest of the statement ("N cannot be selected") is negated to become the sufficient condition. Because the rest is already negative, adding a negation to it will produce a double negation, which cancels out. The final form is: "If N is in then L is in," or, as a diagram:

N :arrow: L

So L being in does NOT kick P out. N requires L, but not vice versa.

Robert Carroll

Get the most out of your LSAT Prep Plus subscription.

Analyze and track your performance with our Testing and Analytics Package.